Antipartículas, conjugación de carga y quiralidad

(¿Por qué / cómo) las antipartículas y los conjugados de carga son cosas diferentes?

Estoy tratando de comprender el efecto de las simetrías discretas en los campos de espinores (neutrinos en particular). En el artículo Dirac, Majorana y Weyl fermiones (sección 7, pág. 25), el autor destaca la distinción entre

a) la operación 'Lorentz-covariante conjugada' , (LCC), dada por ψ ^ = γ 0 C ψ

b) la operación de conjugación de carga , dada por C ψ C 1 = η C ψ ^ .

(Aquí ψ ( X ) describe un campo de fermiones, η C es una fase compleja y C satisface C 1 γ m C = γ m T .)

Por lo que puedo decir, la conjugación covariante de Lorentz reemplaza todas las partículas con sus antipartículas, mientras que la conjugación de carga cambia el signo de todas las cargas asociadas con la partícula. Sin embargo, no estoy seguro de entender la diferencia.

La distinción es importante en el caso de los campos quirales ( PAG L / R ψ = 1 ± γ 5 ψ = ψ L / R ) porque tenemos:

C ψ L C 1 = η C ψ ^ L
pero
ψ L ^ = ψ ^ R .
En otras palabras, el estado antipartícula de un estado quiral izquierdo es quiral derecho, pero el estado conjugado de carga de un estado quiral izquierdo es quiral izquierdo. Hay una declaración de apoyo en la página 26:

...se puede ver mejor con los campos de Weyl para los cuales la quiralidad es lo mismo que la helicidad. La helicidad implica giro y momento, ninguno de los cuales cambia bajo la conjugación de carga. Por lo tanto, la helicidad no se ve afectada por la conjugación de carga, y lo mismo debe ocurrir con la quiralidad.

Toda la manipulación del operador parece razonable, pero estoy confundido porque las definiciones originales solo difieren en una fase compleja, por lo que es difícil entender de dónde proviene la diferencia. ¿Por qué no puedo escribir?

C ψ L C 1 = η C ψ L ^ = η C ψ ^ R
lo que implica que la conjugación de carga cambia la quiralidad?

No puedo encontrar muchos ejemplos de estos conceptos presentados por separado de esta manera. Creo que esta pregunta aborda un problema similar pero no tiene respuesta. Esta respuesta sugiere que mi problema podría estar relacionado con la definición de antipartícula.

Respuestas (1)

Francamente, encuentro este llamado artículo pedagógico bastante ininteligible y no veo lo que el autor quería decir sobre estas dos operaciones. Tampoco puedo entender la "derivación" de 7.3 basada en que la proyección quiral es una "matriz numérica" ​​y, por lo tanto, conmuta con el operador de conjugación de carga.

Más aún el comentario:

Incluso se hacen declaraciones elaboradas en el sentido de que la conjugación de carga cambia la quiralidad. Esto no tiene ningún sentido y se puede ver mejor con los campos de Weyl para los que la quiralidad es lo mismo que la helicidad. La helicidad implica giro y momento, ninguno de los cuales cambia bajo la conjugación de carga. Por lo tanto, la helicidad no se ve afectada por la conjugación de carga, y lo mismo debe ocurrir con la quiralidad.

es engañoso también. Es cierto que el espinor de Weyl zurdo corresponde a la partícula de helicidad izquierda. Pero también describe una antipartícula de helicidad derecha (¡que el propio autor señala antes!) Por ejemplo, durante mucho tiempo no sabíamos que el neutrino tiene una masa y, por lo tanto, en principio podríamos describirlo mediante un solo espinor de Weyl. Pero además del neutrino de helicidad izquierda, el mismo espinor describe el antineutrino de helicidad derecha.

No puede redefinirlo porque la helicidad corresponde a un valor específico del momento angular y, por lo tanto, es observable.

La conjugación de carga transforma la partícula en la antipartícula con la misma helicidad. ¡No puede obtener la antipartícula de helicidad izquierda del espinor de Weyl puramente zurdo! Por lo tanto, no sorprende que la conjugación de carga convierta el espinor de Weyl zurdo en diestro y viceversa.

Este giro también es la razón por la cual el término de interacción débil i ψ L γ 0 γ m W m ψ L no es invariante bajo C transformar. La reflexión espacial también cambia la quiralidad y, por lo tanto, pasa a ser invariante bajo la combinación C PAG .

Para el fermión de Dirac, todo funciona perfectamente para una definición sencilla de la conjugación de carga que denota como ψ ^ . En el espacio de Fock esto conduce a C a s ( pag ) C 1 = b s ( pag ) dónde a s y b s son operadores de creación para partículas y antipartículas respectivamente con una helicidad s . Como era de esperar en el límite sin masa, la a s y b s actuar sobre diferentes componentes quirales por separado mientras a s y b s en el mismo. Entonces, como dije, la conjugación de carga cambia la quiralidad.

También funciona perfectamente para un fermión de Majorana entendido como un espinor de Dirac con una condición de realidad ψ = ψ ^ . La condición de realidad conduce a la identificación a s ( pag ) = b s ( pag ) que es exactamente que la partícula Majorana es lo mismo que una antipartícula con la misma helicidad. Ya no se puede hablar de quiralidad incluso en el caso sin masa porque vincula el componente derecho al izquierdo por definición.

En resumen, no tengo idea de lo que el autor de este artículo trató de decir. Todo funciona bien para una definición directa que todos usan y cambia la quiralidad.

ACTUALIZAR

Tenga en cuenta que antes discutimos solo la quiralidad de los campos pero no los estados.

La forma común de derivar helicidad=quiralidad en el caso sin masa se realiza simplemente para la ecuación de Dirac. El operador helicidad está dado por pag mi σ . La ecuación de Weyl toma la forma,

pag σ ψ L , R = ± mi ψ L , R
dividiéndolo en mi obtenemos que los autoestados de helicidad coinciden exactamente con los autoestados de quiralidad. Podemos concluir que el operador de helicidad es igual a γ 5 es decir, quiralidad.

Cualquier solución restringida a la componente izquierda obtendrá helicidad izquierda. Sin embargo, las soluciones asociadas con antipartículas tienen energías negativas. Se reinterpretan de tal manera que su energía física, momento y giro (y por tanto helicidad) cambian de signo. En la QFT esto se logra afirmando que ψ El operador contiene el operador de aniquilación para la partícula pero el operador de creación para la antipartícula. Porque en el caso de los fermiones, los operadores anticonmutan, por ejemplo, en el operador de espín,

S ^ k = 1 2 d 4 X : ψ ( σ k 0 0 σ k ) ψ :
obtenemos que la contribución de la antipartícula tiene el signo correcto (es decir, opuesto al ingenuo).

Considerando esto si definimos el operador de quiralidad como d 4 X : ψ γ 5 ψ : coincidirá con el operador helicity también en QFT. Entonces deberíamos asignar quiralidad derecha a la antipartícula descrita por el espinor de Weyl zurdo y no cambiará bajo C

En resumen. La quiralidad del operador de campo está cambiando. La quiralidad del estado no lo es.

¡Gracias por la respuesta! Mi problema es que todos (usted, el artículo vinculado, varios otros que respondieron) parecen estar de acuerdo en que esto es sencillo, pero terminan diciendo cosas ligeramente diferentes. Te vincularé a algunas respuestas contradictorias si tengo tiempo más tarde.
@knzhou Ok, investigar esas contradicciones seguramente será divertido y probablemente instructivo.
Bien, comencemos con el artículo de Wikipedia , que establece que C "no altera la quiralidad de las partículas. Un neutrino zurdo se convertiría por conjugación de carga en un antineutrino zurdo". ¿Están combinando quiralidad y helicidad aquí?
Además: ¿cómo llamarías a la quiralidad de una antipartícula de helicidad derecha que surge de un espinor de Weyl zurdo? Ahora que lo estoy pensando, esta es la fuente principal de las contradicciones: diferentes personas lo llaman diestro o zurdo.
@knzhou Sí, parece que ese es el quid del problema. Si bien la helicidad está relacionada con el espín y el momento, la quiralidad se define en términos del espinor que usamos para describir esa partícula y no el estado de (anti)partícula en sí. El operador helicidad es \frac{\vec{p}}{|p}\dot\vec{S} , el operador de quiralidad es γ 5 . Si los aplica al estado de antipartícula proveniente del espinor quiral, obtendrá signos opuestos. Eso, por supuesto, hace que la quiralidad sea una cuestión de convención sobre qué espinor estamos usando para describir la partícula.
@knzhou "Eso, por supuesto, hace que la quiralidad sea una cuestión de convención sobre qué spinor estamos usando para describir la partícula". Más exactamente, para describir el par partícula-antipartícula.
@knzhou Después de repensar la pregunta nuevamente, agrego algo sobre la quiralidad de la antipartícula